Está en la página 1de 2

Errata for A&J Study Manual for exam FM/2

3rd Edition
Errata for A&J Study Manual 3rd Edition SOA Exam FM/ CAS Exam 2

1. Page 48, the solution for Q10, second line: the lower limit of the integration should be from
n instead of 0.

2. Page 67, Solution 2.8, the following figures should be corrected:

s8  14,979.55 instead of 13,643.26 .


1, 200

 14,979.55 
AV7     13, 638.80
 1.098304 

In the last 5th row of the solution, it should be “He has a total of $18,891.19 at t = 7”.

The final answer should be $133,539.34 – $18,891.19 = $114,648.15

3. Page 87, Example 2.20, 2nd line: Starting at t = 23 instead of 22.

4. Page 88, Solution 2.20, under the Second Block: Level Annuity, there are 12 payments
during those periods instead of 11. The present value should be PV  10v10 a
11  54.41 . This
causes some changes to the final answer: PV  41.34  54.41  14.94  20.41  131.10

Copyright © 2010 A&J, 3rd Edition |


2

También podría gustarte